Last visit was: 26 Apr 2024, 21:31 It is currently 26 Apr 2024, 21:31

Close
GMAT Club Daily Prep
Thank you for using the timer - this advanced tool can estimate your performance and suggest more practice questions. We have subscribed you to Daily Prep Questions via email.

Customized
for You

we will pick new questions that match your level based on your Timer History

Track
Your Progress

every week, we’ll send you an estimated GMAT score based on your performance

Practice
Pays

we will pick new questions that match your level based on your Timer History
Not interested in getting valuable practice questions and articles delivered to your email? No problem, unsubscribe here.
Close
Request Expert Reply
Confirm Cancel
Math Expert
Joined: 02 Sep 2009
Posts: 92948
Own Kudos [?]: 619267 [0]
Given Kudos: 81609
Send PM
Math Expert
Joined: 02 Sep 2009
Posts: 92948
Own Kudos [?]: 619267 [0]
Given Kudos: 81609
Send PM
Intern
Intern
Joined: 09 Oct 2016
Posts: 7
Own Kudos [?]: 2 [0]
Given Kudos: 0
Send PM
avatar
Intern
Intern
Joined: 26 Apr 2018
Posts: 4
Own Kudos [?]: 0 [0]
Given Kudos: 2
Send PM
Re: S92-16 [#permalink]
Answer E does not necessarily lead to higher sales.
As no information is given about consumer behaviour, one should not make assumptions about consumer preferences regarding Price VS Quality of staples.
In addition, the fact that the superior stapler only works with staples from one company, might require a higher effort when buying staples as you cannot just buy any staple available. Therefore it is not safe to assume that a superior and more expensive product necesarily leads to higher sales.

In a market that is doubling in just three years, it is very likely to achieve an increased sales of staples. Otherwise it means your market share drops drastically and , assuming all variables are constant, no reason is given to assume such a drastic drop in market share. As a result of increased sales of staplers that only function with the staple produced by the same company, the plan enables the company to secure higher revenues for the future.
Therefore answer B is correct in my opinion.
GMAT Club Bot
Re: S92-16 [#permalink]
Moderator:
Math Expert
92948 posts

Powered by phpBB © phpBB Group | Emoji artwork provided by EmojiOne